WBR0862
Author | [[PageAuthor::Rim Halaby, M.D. [1]]] |
---|---|
Exam Type | ExamType::USMLE Step 1 |
Main Category | MainCategory::Pathology |
Sub Category | SubCategory::Pulmonology |
Prompt | [[Prompt::A 56 year old man presents to the neurology clinic for 3 weeks of upper and lower extremity weakness. The patient explains that he works as a plumber and that his everyday chores have become much harder since he has started to notice this weakness. Despite this, he has noticed a pattern that allows him to overcome it. He reports that the longer he is at rest, the weaker he feels, and as soon as he exerts himself he restores a lot of the strength in his extremities. Physical exam reveals 2/5 strength in all four extremities, bilateral eyelid ptosis, and absent deep tendon reflexes. With a certain diagnostic suspicion, the physician orders a chest CT that reveals a 2.5 cm mass adjacent to the left stem bronchus. Biopsy of the mass is likely to reveal which of the following findings?]] |
Answer A | AnswerA::Small dark blue neuroendocrine cells |
Answer A Explanation | AnswerAExp:: |
Answer B | AnswerB::Keratin pearls and retracted cells |
Answer B Explanation | AnswerBExp:: |
Answer C | AnswerC::Pleomorphic giant cells |
Answer C Explanation | AnswerCExp:: |
Answer D | AnswerD::Granuloma formation with caseating necrosis |
Answer D Explanation | AnswerDExp:: |
Answer E | AnswerE::Granuloma formation without caseating necrosis |
Answer E Explanation | AnswerEExp:: |
Right Answer | RightAnswer::A |
Explanation | [[Explanation:: Educational Objective: |
Approved | Approved::No |
Keyword | WBRKeyword::Kulchitsky cells, WBRKeyword::small cell lung carcinoma, WBRKeyword::Lambert Eaton Myasthenic Syndrome, WBRKeyword::LEMS |
Linked Question | Linked:: |
Order in Linked Questions | LinkedOrder:: |